What is the range of the function graphed below?
O (-∞0,4]
O [-4.0) [2,00)
O [-4,00)
O (-4,0] (2,00)

What Is The Range Of The Function Graphed Below?O (-0,4]O [-4.0) [2,00)O [-4,00)O (-4,0] (2,00)

Answers

Answer 1

Answer:       B.     [–4, 0)  ∪  [2, ∞)

To find the range, find out what y-values are possible for the function in the graph.

Symbols

We need to pay attention to what symbols to use.

Reading the diagram

Solid circle

The function touches the point.

Empty circle

The function gets close, but does not touch the point.

Arrow

The function keeps going to infinity.

Writing range in set notation

Square brackets: [ and ]

These show that the function touches the point.

Curved brackets: ( and )

These show that the function gets close but does not touch the point.Infinity always uses a curved bracket.

Union of sets:

This symbol shows that sets of values are talking about the same thing. It's almost like the word 'and'.

Finding the range of the graphed functionLower blue line

Let's start by looking for the lowest possible y-value at the bottom of the diagram. On the graph, the point (4, –4) marked with a solid circle.

The function touches the point and stops. Since the y-value is –4, the lowest possible y-value is:     [tex]-4[/tex]

If we follow the blue line upwards, we find an empty circle at (0, 0). The y-value in (0, 0) is 0. This means that the function gets close, but does not include:     [tex]0[/tex]

So, the first part of our range is:

[–4, 0)

Upper blue line

The blue line on the top starts where y = 2 at a solid circle. So, the function includes the y-value:    [tex]2[/tex]

If we follow this blue line upwards, it ends with an arrow. This means it will keep going to infinity, which is the symbol:    [tex]\infty[/tex]

So, the second part of our range is:

[2, ∞)

Putting it together

Put the lower and upper blue lines together with the union of sets symbol. The range is expressed as:    [–4, 0)  ∪  [2, ∞)

Learn more about domain and range here:

https://brainly.com/question/20349151


Related Questions

Explain how to find the average rage of change for a function over a given interval.

Answers

The average rate of change for a function over an interval is the slope

How to explain the steps?

Take for instance, we have the function to be

Function f(x)

And the interval is (a,b)

The average rate of change for the function over the interval is

Rate = (f(b) - f(a))/(b - a)

The above represents the slope

Take for instance, we have:

f(x) = x^2 [2, 4]

We have:

f(2) = 2^2 = 4

f(4) = 4^2 = 16

So, we have:

Rate = (16 - 4)/(4 - 2)

Rate = 6

Read more about average rate of change at:

https://brainly.com/question/8728504

#SPJ1

Points Z, L, and S are:

Answers

Points Z, L, and S are collinear points

How to determine the relationship?

When 2 or more points are on a line, the points are said to be collinear.

From the figure, we have:

Points Z, L and S on the same line

This means that they are collinear points

Hence, points Z, L, and S are collinear points

Read more about collinear points at:

https://brainly.com/question/1593959

#SPJ1

What is the absolute value of -6 ?

Answers

Answer:

the answer is just 6

Step-by-step explanation:

Answer:

Step-by-step explanation:

An absolute value of a number is written like this | x |, where x could be any number that you want.

The absolute value is a math function that counts from your given number to 0.

Example: If you put the number 3, the function will count from 3 to 0, and the number will be 3.

| 3 | = 3

If you input the number 8, from 8 to 0 are 8 numbers. So, the output's function will be 8.

| 8 | = 8

If you input a negative number, for example, -6, the function will do the same thing. It will count from -6 to 0. From -6 to 0, are 6 numbers.

| -6 | = 6

Any number that you input in this function, will always output the same number, but positive

| -10 | = 10

| -46348 | = 46348

Hope I helped !

If f (x) = one-ninth x minus 2, what is ?
A.
B.
f - 1 Baseline (x) = one-ninth x + 2
f - 1 Baseline (x) = -2 x + one-ninth
Please see picture: helpppppp

Answers

Answer:

1st option

Step-by-step explanation:

let y = f(x) and rearrange making x the subject , that is

y = [tex]\frac{1}{9}[/tex] x - 2 ( add 2 to both sides )

y + 2 = [tex]\frac{1}{9}[/tex] x ( multiply both sides by 9 to clear the fraction )

9(y + 2) = x

9y + 18 = x

change y back into terms of x with x = [tex]f^{-1}[/tex] (x) , then

[tex]f^{-1}[/tex] (x) = 9x + 18

Answer: A

Step-by-step explanation:

GOT IT RIGHT ON EDG TEST

What is the area of the shaded sector of the circle? 9pi units squared 27pi units squared 81pi units squared 162pi units squared

Answers

Answer:

27

Step-by-step explanation:

i promise :)

Answer:

27π units^2

Step-by-step explanation:

Please help me with this thanks!!

Answers

Answer: 47°

Step-by-Step Explanation:

In a Triangle the Sum of all the Angles equal to 180° (Angle Sum Property)

Therefore,
x + 78 + 55 = 180
x + 133 = 180
x = 180 - 133
=> x = 47

Hence, ∠x = 47°

Answer:

x° = 47°

Step-by-step explanation:

The sum of the measurements of three interior Angles in a triangle is 180°.This is known as angle sum property.Same case is here.

78°,55° and x° are interior angles here.

So,

[tex] 78{}^{ \circ} + 55 {}^{ \circ} + x {}^{ \circ} = 180 {}^{ \circ} [/tex]

Solving for x°,

[tex]133 {}^{ \circ} + x {}^{ \circ} = 180 {}^{ \circ} [/tex][tex]x {}^{ \circ} = 180 {}^{ \circ} - 133 {}^{ \circ} [/tex][tex]x {}^{ \circ} = \boxed{47 {}^{ \circ} }[/tex]

Hence,the value of x° is 47°.

Here is a list of numbers: 3.9 , 7 , 3.3 , 1 , 1.4 , 1.7 , 2.1 , 3.3 , 5.2 State the median. Give your answer as a decimal.

Answers

A median.

The "middle" of a sorted list of numbers. To find the Median, place the numbers in value order and find the middle number.

When there are two middle numbers we average them.

We have:

3.9, 7, 3.3, 1, 1.4, 1.7, 2.1, 3.3, 5.2

Let's sort the numbers from the smallest to the largest

1, 1.4, 1.7, 2.1, 3.3, 3.3, 3.9, 5.2, 7

The median is 3.3

Answer:

3.3

Step-by-step explanation:

The first thing you have to do is put them in order from least to greatest.

1, 1.4, 1.7, 2.1, 3.3, 3.3, 3.9, 5.2, 7

Check off one from the left side and one from the right side until you are left with the one in the middle.

When you do that, you are left with 3.3

y = x2 − 2x − 19
y + 4x = 5

Answers

Answer:

(x=4, y=-11) (x=-6, y=29)

Step-by-step explanation:

The solution is in the picture above...

what is an equivalent expression ​

Answers

Answer:

1st option

Step-by-step explanation:

using the rules of logarithms

• log x + log y = log xy

• n logx ⇔[tex]x^{n}[/tex]

given

2[tex]log_{2}[/tex] 3 + [tex]log_{2}[/tex] 3

= [tex]log_{2}[/tex] 3² + [tex]log_{2}[/tex] 3

= [tex]log_{2}[/tex] 9 + [tex]log_{2}[/tex] 3

= [tex]log_{2}[/tex] (9 × 3)

= [tex]log_{2}[/tex] 27

whenx=2y=50 and whenx=4y=100 Which direct variation equation can be used to model this function?

Answers

The equation which is used to model this function is y = 25x.

What is equation?

An equation is a mathematical expression that contains an equals symbol. Equations often contain algebra.

Given:

x=2 , y=50

x=4, y=100

As, corresponding to x =2 , y =50

For x=1 , y=25

Hence, the equation which is used to model this function is y = 25x.

Learn more about equation here:

https://brainly.com/question/10413253

#SPJ1

Suppose a deposit of $ 2 , 000 in a savings account that paid an annual interest rate r (compounded yearly) is worth $ 2 , 209 after 2 years. using the formula a = p ( 1 + r ) t , we have 2 , 209 = 2 , 000 ( 1 + r ) 2 solve for r to find the annual interest rate (to the nearest tenth).

Answers

Answer:

5.1% (nearest tenth)

Step-by-step explanation:

Annual Compound Interest Formula

[tex]\large \text{$ \sf A=P\left(1+r\right)^{t} $}[/tex]

where:

A = final amountP = principal amountr = interest rate (in decimal form)t = time (in years)

Given:

A = $2,209P = $2,000t = 2 years

Substitute the given values into the formula and solve for r:

[tex]\implies \sf 2209=2000\left(1+r\right)^{2}[/tex]

[tex]\implies \sf \dfrac{2209}{2000}=(1+r)^2[/tex]

[tex]\implies \sf 1.1045=(1+r)^2[/tex]

[tex]\implies \sf \sqrt{1.1045}=1+r[/tex]

[tex]\implies \sf r = \sqrt{1.1045}-1[/tex]

[tex]\implies \sf r = 0.05095194942...[/tex]

[tex]\implies \sf r = 5.095194942...\%[/tex]

Therefore, the annual interest rate is 5.1% (nearest tenth)

Please help i dont get this at all so please i will highly appreciate it .Thank youuu!!!

Answers

Answer:

4y^4

Step-by-step explanation:

This polynomial is supposed to be ordered:

-3x^4+4y^4+5x^2y^2-10xy^3+9x^3y

I dont totally understand the question but I hope the ordered polynomial helps

Explain what the result of your proof tells you about angles a and b. Specifically, if you measured one angle, what would you know about the other? (2 points)

Answers

If the measure of one of the angles ∠a or ∠b is known, then the other angle is given by subtracting the known angle from 180°

What are supplementary angles?

Two angles whose sum is 180° are called supplementary angles. If a straight line is intersected by a line, then there are two angles form on each of the sides of the considered straight line.

The required two-column proof is presented as follows;

Statement                                                    Reason

∠1 and ∠a are linear pair angles                Given

∠1 + ∠a = 180°                                              Linear pair ∠s are supplementary

∠2 and ∠b are linear pair angles               Given

∠2 + ∠b = 180°                                             Linear pair ∠s are supplementary

∠a and ∠b are consecutive interior ∠s      Definition

x║y                                                               Given

∠1 and ∠b are corresponding angles        Definition

∠1 ≅ ∠b                                                        Corr. ∠s formed between║ lines

∠1 = ∠b                                                         Definition of congruency

∠a + ∠b = 180°                                              Substitution property

∠a and ∠b are supplementary                    Definition

We can conclude from the proof that given ∠1 and ∠b are located in corresponding locations relative to the common transversal of the parallel lines, they (∠1 and ∠b) are equal.

Therefore, the sum of ∠a and ∠b is 180°, given that the sum of ∠1 and ∠a is 180° by substitution property of equality. So, ∠a and ∠b are supplementary angles.

Therefore, If the measure of one of the angles ∠a or ∠b is known, then the other angle is given by subtracting the known angle from 180°

Learn more about supplementary angles here:

https://brainly.com/question/2882938

#SPJ1

15.95 to the nearest cent

Answers

Step-by-step explanation:

95 95 branded to the nearest 10 with the number line

deter determine the two constructive multiples of 10 that bracket 95

9 is between 90 and 100

asteroid on the number line 95 is the midpoint between the 90 and 100

therefore 95 rounding to the nearest 10 is equal to 100

16
because 95 rounds to 100 which makes 15.95 into 16

Please help meeeeeeeeee???

Answers

The answer for the exponential expressions 1,2,3 and 4 will be [tex]4.5,\frac{1}{2},2,1[/tex] respectively.

What exactly is an exponent?

Exponential notation is a type of mathematical shorthand that allows us to write complex formulas in a more concise manner.

The basis of an exponent is a number or letter. It means that the base will be raised to a given level of power. The base is X, and the power is n.

The properties of the exponent used;

[tex]\rm m^a \times m^b = m^{a+b} \\\\ m^a \times n^a = {mn}^a \\\\ m^a \times m^b = m^{a-b} \\\\ m^0 = 1[/tex]

Expression 1;

[tex]\frac{(2.3^{-2})^2(5.3^2.2)^2}{(3)^-2(5.2)^2} \\\\ \frac{2^3.3^{-6}.5^2.3^6}{3^{-2}.5^2.2^2} \\\\ 4.5[/tex]

Expression 2;

[tex](3^3)(4^0)(3.2)^{-3}(2)^2 \\\\ 3^3.1.3^{-3}.2^{-3}.2^2 \\\\ \frac{1}{2}[/tex]

Expression 3;

[tex]\frac{(3^7.4^7)(2.5)^{-3}(5)^2}{12^7.5^{-1}.2^{-4}} \\\\ \frac{3^7.4^7.25^{-3}.5^{-3}.5^2}{12^7.5^{-1}2^{-4}} \\\\ 2[/tex]

Expression 4;

[tex]\frac{(2.3)^{-1}.2^0}{(2.3)^{-1}}\\\\ \frac{2^{-1}.3^{-1}.1}{2.3}\\\\ 1[/tex]

Hence, the answer for the exponential expressions 1,2,3, and 4 will be [tex]4.5,\frac{1}{2},2,1[/tex] respectively.

To learn more about exponents, see

https://brainly.com/question/5497425

#SPJ1

How do you find what exponents of i are? like I know i^2 is -1, but how are you supposed to figure out i^26? do you have to write it all out?

THANK YOU

Answers

Answer:

-1

Step-by-step explanation:

so "i" is the square root of -1 (√-1). We know that i² = (√-1)², and that when we square a square root we end back up with whatever is inside of the square root.

This is why i² = -1

We can find a pattern from writing out a few exponents:

i¹ = i

i² = -1

i³ = -i  [(-1)(i)]

i⁴ = 1  [-1 · -1]

------

i⁵ = i  [(1)(i)]

i⁶ = -1 [(i)(i)]

i⁷ = -i [(-1)(i)]

i⁸ = 1 [(-i)(i)= -(i²)= -(-1) = 1]

------

i⁹ = i [1 · i]

we can see that the pattern of i repeats every 4 numbers, so by dividing 26 by 4 (and finding the remainder), we can figure out which step of the pattern we are on

26 / 4 = 6 [remainder of 2]

we know that there is a remainder of 2, meaning that we are on the "second step" in the pattern: -1

you can do the same thing for any number

hope this helps!!

Answer:

-1

Step-by-step explanation:

4 goes into 26 6 times, and then there's 2 remainder, so it's the same thing as i², which is -1

hope this is helpful :)

Help with Pre Calculus

Answers

the answer is the first one .. y=3x-4

Find (gof)(3)
f(x) = x + 21
g(x) = -x²
a. -25
C.
4
b. 1
d. -15
Please select the best answer from the choices provided

Answers

A function assigns the values. The value of (gof)(3) is -576.

What is a Function?

A function assigns the value of each element of one set to the other specific element of another set.

Given f(x)=x+21 and g(x)=-x², therefore, the value of (gof)(x) will be,

(gof)(x) = g(f(x))

           = -(x+21)²

The value of (gof)(3) will be,

(gof)(3) = -(3+21)²

            = -576

Hence, the value of (gof)(3) is -576.

Learn more about Function:

https://brainly.com/question/5245372

#SPJ1

How many sulotions does this system have? 6x-y=-1 6x+y=-1

Answers

Answer:

6/6

Step-by-step explanation:

6/6 not equal -1/1 so these lines intersect each other at only one point hence these have only one solution.

Answer:

I can give you the answer if you can answer a question

Step-by-step explanation:

Which table can be created using the equation below?
-2 + 4x = y
X
y
-5-22
0
3
X
y
-22 -5
-2 0
10
3
X
-5
0
-2
10
03
y
-18
-2
10

Answers

Answer:

h

Step-by-step explanation:

h

what is the answr to -5(b + 1) = 8

Answers

[tex] - 5(b + 1) = 8 \\ - 5b - 5 = 8 \\ - 5b = 8 + 5 \\ - 5b = 13 \\ b = \frac{ - 13}{5} [/tex]

PLEASE GIVE BRAINLIEST

\\\///\\\///\\\///\\\///\\\///\\\///\\///\\\///\\\///\\\///\\\///\\\//\\//\\//\\//\\\///\\//\\\///\\\///\\\///\

                                      [tex]\Rsh[/tex] Hey! [tex]\Lsh[/tex]

[tex]\Rsh[/tex]Let's solve this! We can do this! [tex]\Lsh[/tex]

>>> First, let's multiply -5 by b and 1. Then we obtain

[tex]\pmb{-5b-5=8}[/tex].

>>> Now we add 5 to both sides.

[tex]\pmb{-5b=13}[/tex]

>>> And then, divide both sides by -5.

[tex]\pmb{b=-\dfrac{13}{5}}[/tex]

>>> That is the value of b.

><><><><><<><><><><><><><><><><><><><><><><><><><><><><><><><><>

Hope the answer - and explanation - assisted you!

Happy studies! :)

A company manufactures school desks. The table shows the number of units that the company sold each month over a nine-month period
beginning in January.
Month, x
Units, y
Which equation best models the situation?
1
800
2
500
3
400
4
300
5
200
6
300
7
450
8
600
9
800

Answers

Based on the values of x and y for the company and its school desks, the equation that best models the situation is D. y = 34x² - 334x + 1,075.

How does this equation model the situation?

The best equation would be the one that gives the closest value of y to the value of the actual value of y on the table.

The first two equations can be ruled out because the table shows that this is not a linear model as there is no constant rate of change.

Test the other two equations:

x=1:

y = -34(1)² + 334(1) + 1,075

= 1,075

This is too high.

Other equation:

y = 34(1)² - 334(1) + 1,075

= 775

This is close to 800 so this is the best equation.

Find out more problems about the best models for situations at https://brainly.com/question/27157349.

#SPJ1

at what height will the air pressure equal 50%

Answers

Answer:

Im pretty sure its C which is 5776.2m

Step-by-step explanation:

The World Health Organization lists the following overall literacy rates per 100 people for countries. Which answer choice includes all countries with literacy rates above 90%?

Answers

Answer:

The answer is D

Cuba, Poland, U.S., Argentina, Turkey, and Spain

Step-by-step explanation:

Basically, what it was asking, is what countries are over 90%? So, all you have to do - to put it simply - is just look at the countries that's numbers are greater than 90%

Which equation is the inverse of y = 9x² - 4?​

Answers

Answer:

y = ± [ √(x+4) ] / 3

Step-by-step explanation:

1) To find the inverse of the function y = 9x2 – 4, exchange the variables x and y, and then solve for the new y.

2) Exchange of the variables: y = 9x^2 – 4 → x = 9y^2 – 4

3) Solve for y:

=> 9y^2 = x + 4

=> y^2=> y = ± [ √(x + 4) ] / 9

= > y = ± [ √(x + 4) ] / 3

This is the last question PLSSSSS HELP

Answers

Answer:

D

Step-by-step explanation:

The volume of a cylinder is pi * r^2 * h

A) V = 3.14 * (0.7/2)^2 * 6.2 = 2.4 m^2

B) V = 3.14 * (0.9/2)^2 * 3.4 = 2.2 m^2

C) V = 3.14 * (1.2/2)^2 * 6.8 = 7.7 m^2

D) V = 3.14 * (1.4/2)^2 * 6.2 = 9.5 m^2

3.5 decimals to mixed numbers

Answers

1 2/3
It’s one and two thirds written out

A local mechanic on average services 14 cars a week. How many cars does he service in 4 years

Answers

If he does 14 per week
He would do 14 * 4 = 56 per month
He would do 56 * 12 = 672 per year
In four years, he would service 672 * 4 = 2,688
There are 52.1 weeks in a year so i he does 14 cars a week. In 4 years there are (52.1 x 4) 208.4 weeks. So 208.4 multipled by 14 and the answer is 2917.6

Select the correct answer.
Which graph best models the inequality?
y ≤ -2/5x + 2

Answers

Answer:

it would look like this

Step-by-step explanation:

The price of a computer at an electronics store is $1200. If the store gives an "End of the year Sale"
discount of 20%, how much Mark needs to pay to buy the computer?

Answers

Answer
$960

Explanation
1,200 × 20 ÷ 100 = 240
1,200 - 240 = 960

Answer:

$960

Step-by-step explanation:

The discount is 20%, so we need to find 80% of the original price:

[tex].80 \times 1200 = 960[/tex]

Other Questions
Please help fast!!!Multiply and reduce to lowest terms. Convert into a mixed number if necessary.5 x 2/3 = Consider the table that includes the input and output values of a function.Which best describes the constant difference for the function?a constant second difference of 6a constant second difference of 8a constant third difference of 6a constant third difference of 8 The nurse prepares a 7 year old influenza injection. The nurse explains that the client will receive "medicine under the skin" and the client is visibly anxious. Which nursing intervention is appropriate? A direct democracy is a form of government in which citizensAnswer:vote on all laws and important decisions.Hope this helped The maximum number of turning points of a 9th degree polynomial is? In times of reduced visibility or at night, what color all-around light should a non-powered canoe or kayak display?. Which is required for both anaerobic respiration and aerobic respiration?O oxygenO water mitochondriaglucose At the __________ stage in the personal selling process, a salesperson's physical appearance, speech habits, personality, and even hygiene will have the greatest effect. Please keep quiet,I......(listen) to the radio.You know I.....(listen)to the news in the mornings. The area of a rectangle is 44m2 , and the length of the rectangle is 3m less than double the width. Find the dimensions of the rectangle. B) Rewrite the sentences in the interrogative.1. They prepare dinner.Do they prepare dinner?2. Sally and Alan like broccoli.3. David drinks lemonade.Do Sally and alan like tomatoes?_Does David drinks lemonade? What does heterozygous describe? (2 points) aA gene that is not expressed when paired with a dominant allele. bA trait produced from two different alleles. cA trait produced from two alleles that are alike. dA trait hidden by another trait. I need help with number 10 please Which two themes does Gaines develop in A Lesson Before Dying?O By thinking for oneself, a person can overcome problems.O It takes practice to learn when to fight and when to give up.O People often lie to themselves to justify their own behavior. One can both learn from the past and move forward from it. Which of the following is NOT an example of a way to help reduce mental health stigmas? A. Avoiding judgement of people with mental illness before you get to know them B. Becoming more comfortable with the use of words like "crazy," "nuts," or "insane" C. Criticizing any negative portrayal in the media of those living with mental illness D. Exercising compassionate patience when around those suffering from mental illness A 0.2 kg ball is attached to a string and whirled around in a circle overhead. The string breaks if the force on it exceeds 80 N. What is the maximum speed the ball can have when the radius of the circle is 1m? What is the hypothesis? What educated guess are the scientists making?Please help with this question about the beet root extract The framers wanted to limit federal powers. Match the key ideas they considered important to the terms they used Which word from the excerpt is a domain word?balancekartsweeksteachers How were the beliefs of the nation of islam different from the beliefs of southern christian leadership conference?